Đến nội dung

triethuynhmath nội dung

Có 1000 mục bởi triethuynhmath (Tìm giới hạn từ 09-06-2020)



Sắp theo                Sắp xếp  

#387399 Vòng 1 BMO 2013

Đã gửi bởi triethuynhmath on 17-01-2013 - 14:55 trong Thi HSG Quốc gia và Quốc tế

Câu 3:Giải hệ phương trình
$\left\{\begin{matrix} x^{2}-4y+7=0\\ y^{2}-6z+14=0\\ z^{2}-2x-7=0 \end{matrix}\right.$

Cho mình hỏi $BMO$ là gì vậy bạn????
Câu này chắc dễ nhất đề.
Cộng theo vế ba phương trình của hệ ta được: $x^2-2x+y^2-4y+z^2-6z+14=0\Leftrightarrow (x-1)^2+(y-2)^2+(z-3)^2=0\Leftrightarrow x=1,y=2,z=3$
XOng nhớ thử lại nghiệm.



#387303 Giải hệ : $\left\{ \begin{array}{l...

Đã gửi bởi triethuynhmath on 16-01-2013 - 23:22 trong Phương trình, hệ phương trình và bất phương trình

Giải hệ phương trình:$$\left\{ \begin{array}{l}
x + y - z = 7 \\
{x^2} + {y^2} - {z^2} = 37 \\
{x^3} + {y^3} - {z^3} = 1 \\
\end{array} \right.$$
___
NLT

Dạng hệ này cho THCS có vẻ vẫn còn khá khó đó :)
Ta có đẳng thức quen thuộc:
$(x+y-z)^3-(x^3+y^3-z^3)=3(x+y)(y-z)(x-z)=7^3-1=342\Leftrightarrow (z+7)(7-x)(7-y)=114\Leftrightarrow (z+7)(49-7(x+y)+xy)=114\Leftrightarrow (z+7)(49-7(7+z)+xy)=114(1)$
Mặt khác từ 2 phương trình đầu cho ta:
$(x+y)^2-z^2-37=2xy\Leftrightarrow (z+7)^2-z^2-37=2xy\Leftrightarrow 14z+12=2xy\Leftrightarrow 7z+6=xy$
Đem thay vào $(1)$ ta được: $(z+7)(49-49-7z+7z+6)=114\Leftrightarrow 6(z+7)=114\Leftrightarrow z=12$
Đến đây...



#387174 Tìm giá trị lớn nhất của: $(xy+wz)^2$

Đã gửi bởi triethuynhmath on 16-01-2013 - 16:31 trong Bất đẳng thức - Cực trị

Cho $x,y,z,w$ là các số thực thỏa mãn:
$x^2+y^2-\frac{xy}{2}=w^2+z^2+\frac{wz}{2}=36,xz+yw=30$
Tìm giá trị lớn nhất của: $(xy+wz)^2$



#387173 $ \sqrt{y-a^2}+\sqrt{z-a^2}=1 ...$

Đã gửi bởi triethuynhmath on 16-01-2013 - 16:28 trong Phương trình - Hệ phương trình - Bất phương trình

Cho $a,b,c \geq 0$ và $a+b+c= \frac{\sqrt{3}}{2}$. Chứng minh rằng hệ phương trình sau có một nghiệm duy nhất:
$\left\{\begin{matrix} \sqrt{y-a^2}+\sqrt{z-a^2}=1 \\ \sqrt{x-b^2}+\sqrt{z-b^2}=1 \\ \sqrt{x-c^2}+\sqrt{y-c^2}=1 \end{matrix}\right.$



#387172 $\sum \frac{1}{(1+x)^3}+\frac{5...

Đã gửi bởi triethuynhmath on 16-01-2013 - 16:23 trong Bất đẳng thức - Cực trị

Cho $x,y,z >0$,$xyz=1$. CHứng minh rằng:
$\sum \frac{1}{(1+x)^3}+\frac{5}{(1+x)(1+y)(1+z)}\geq 1$



#387170 Tìm số ngày nhiều nhất của kỳ nghỉ

Đã gửi bởi triethuynhmath on 16-01-2013 - 16:12 trong Tổ hợp và rời rạc

Do sau một thời gian không có lời giải nên mình sẽ post ý tưởng chính để giải bài này.
Để làm bài này ta sẽ chia thành 2 phần:
Phần 1: Chứng minh kì nghỉ có không quá $5$ ngày.
Chứng minh: Xét kì nghỉ đã trải qua $4$ ngày. Ta thấy do kì nghỉ đã trôi qua $4$ ngày nên $2$ căn phòng ở hai biên đã được ở $8$ lần (Mỗi phòng 4 lần). Vậy có tối đa $8$ người đã ở phòng ấy.Vậy chắc chắn có $1$ người chưa ở biên bao giờ. Vậy 4 ngày qua họ đều nằm ở các phòng giữa. Vậy họ đã ở cạnh $8$ người nên vào ngày thứ 5 họ phải ở biên và cạnh người còn lại họ chưa ở cùng. Như vậy là nhóm người không thể tiếp tục ở khách sạn được nữa. Vậy số ngày của kì nghỉ không quá $5$.
Phần 2: Đó là ta đưa ra một trường hợp mà kì nghỉ có thể kéo dài đến $5$ ngày, nghĩa là ta sắp xếp lịch ở khách sạn cho nhóm người trong vòng $5$ ngày. Phần này thì không phải là dễ và tương đối phức tạp, mọi người nghĩ thử xem :)



#387162 Tìm GTNN của $A=\sqrt{1+a^{4}} +\sqrt...

Đã gửi bởi triethuynhmath on 16-01-2013 - 15:31 trong Bất đẳng thức và cực trị

Tìm GTNN của $A=\sqrt{1+a^{4}} +\sqrt{1+b^{4}}$ với (1+a)(1+b)=$\frac{9}{4}$

Mình nghĩ bài này nên cho $a,b \geq 0$ chứ nhỉ :-?Áp dụng Bunyakovsky,ta có: $A\sqrt{5}=\sqrt{5(a^4+1)}+\sqrt{5(b^4+1)}=\sqrt{(1^2+2^2)(a^4+1)}+\sqrt{(2^2+1^2)(b^4+1)}\geq a^2+2+b^2+2\geq \frac{(a+b)^2}{2}+4$
Mặt khác: $\frac{9}{4}=(1+a)(1+b)\leq \frac{(2+a+b)^2}{4}\Rightarrow (a+b+2)^2\geq 9\Rightarrow (a+b+2)\geq 3\Rightarrow a+b \geq 1\Rightarrow a^2+b^2 \geq \frac{1}{2}$
Kết hợp với trên ta tìm được Min của biểu thức :)



#387063 C/m O là trung điểm MN

Đã gửi bởi triethuynhmath on 15-01-2013 - 22:41 trong Hình học

Từ điểm S ở ngoài (O),vẽ tiếp tuyến SD,cát tuyến SAB(D là tiếp điểm) với (O).Vẽ đường kính DC của (O),tia SO cắt hai tia CA,CB theo thứ tự tại M,N.CM:O là trung điểm của MN.

Gợi ý cho bạn xíu nhé:
Từ $A$ vẽ đường thẳng $// MN$ cắt $CD$ tại $I$ và cắt $BC$ tại $J$. Lấy $K$ là trung điểm $AB$. Ta sẽ chứng minh $I$ là trung điểm $AJ$ bằng cách chứng minh $IK//BJ$ Ý tưởng tự nhiên nhất là chứng minh tứ giác: $IKDA$ nội tiếp bằng cách chứng minh: $\angle KAI=\angle KDI$ thông qua $AJ//SN$ và $SDKO$ là tứ giác nội tiếp :)



#387057 $P(x^2 -2002) \vdots P(x)$

Đã gửi bởi triethuynhmath on 15-01-2013 - 22:22 trong Đa thức

Có tồn tại hay không đa thức P(x) thỏa:
(1) $degP(x) = 2003$
(2) $P(x^2 -2002) \vdots P(x)$ ($\forall x \in \mathbb{R}$)


ps: hồi chiều làm kiểm tra bị bể bài này :((((

Có tồn tại đa thức này và ta sẽ chỉ ra đa thức đó là đa thức: $(x+\frac{1}{2}(\sqrt{8009}-1))^{2003}$
Chứng minh đa thức này thỏa mãn: $P(x^2-2002)=(x^2-2002+\frac{1}{2}(\sqrt{8009}-1))^{2003} \vdots (x+\frac{1}{2}(\sqrt{8009}-1))^2003$
Áp dụng định Lý Bezeout. Ta có: Xét đa thức $Q(x)=x^2-2002+\frac{1}{2}(\sqrt{8009}-1)$
Ta nhận thấy $Q(-\frac{1}{2}(\sqrt{8009}-1))=\frac{1}{4}(\sqrt{8009}-1)^2-2002+\frac{1}{2}(\sqrt{8009}-1)=0\Rightarrow (x^2-2002+\frac{1}{2}(\sqrt{8009}-1))\vdots (x+\frac{1}{2}(\sqrt{8009}-1))$.Vậy ta có đpcm :)



#386790 $\sum \frac{a}{b+2c+3d}\geq \fra...

Đã gửi bởi triethuynhmath on 14-01-2013 - 21:44 trong Bất đẳng thức và cực trị

Mình post ảnh xin mọi người thông cảm nhé :lol: !

Bài này là $Cauchy-Schwarz$.
$\sum \frac{a}{b+2c+3d}\geq \frac{(a+b+c+d)^2}{4(ab+ac+ad+bc+cd+bd)}$
Cuối cùng ta đưa về chứng minh: $3(a^2+b^2+c^2+d^2)\geq 2(ab+ac+bc+cd+ad+bd)$
Tới đây $Cauchy$ phát :D



#386783 $\sum \frac{a^{4}}{b+c}\geq...

Đã gửi bởi triethuynhmath on 14-01-2013 - 21:32 trong Bất đẳng thức và cực trị

Tiếp tục nhé các bạn!
Bài 17: Cho a, b, c > 0.CMR
$\sum \frac{a^{4}}{b+c}\geq \frac{\sum a^{3}}{2}$

Bất đẳng thức Chebyshev cái ra ngay.
Giả sử $a \geq b \geq c$ nên: $\frac{a}{b+c}\geq \frac{b}{c+a} \geq \frac{c}{a+b}\Rightarrow \sum \frac{a^4}{b+c}\geq \frac{1}{3}(a^3+b^3+c^3)\sum \frac{a}{b+c}\geq \frac{a^3+b^3+c^3}{2}(Nesbit)$



#386764 Giải Hệ Phương trình: $ 3-\frac{5}{y+42x} \sqrt{2y}=4 ...$

Đã gửi bởi triethuynhmath on 14-01-2013 - 21:01 trong Phương trình - Hệ phương trình - Bất phương trình

$\left\{\begin{matrix} \left ( 3-\frac{5}{y+42x} \right )\sqrt{2y}=4 & \\ \left ( 3+\frac{5}{y+42x} \right )\sqrt{x}=2 & \end{matrix}\right.$

Dạng hệ này thì đúng là khó thật.
Dễ thấy $x,y >0$
Hệ đã cho tương đương: $\left\{\begin{matrix} 3-\frac{5}{y+42x}=\frac{4}{\sqrt{2y}} \\ 3+\frac{5}{y+42x}=\frac{2}{\sqrt{x}} \end{matrix}\right.\Rightarrow \left\{\begin{matrix} \frac{2}{\sqrt{2y}}+\frac{1}{\sqrt{x}}=3 \\ \frac{1}{\sqrt{x}}-\frac{2}{\sqrt{2y}}=\frac{5}{42x+y} \end{matrix}\right.\Rightarrow \frac{1}{x}-\frac{2}{y}=\frac{15}{42x+y}\Leftrightarrow \frac{y-2x}{xy}=\frac{15}{y+42x}\Leftrightarrow (y-2x)(y+42x)=15xy\Leftrightarrow xy+y^2+40xy-84x^2=15xy\Leftrightarrow 84x^2-y^2-26xy=0\Leftrightarrow ...$



#386571 $\frac{a^{4}+b^{4}+c^{4}+d^...

Đã gửi bởi triethuynhmath on 13-01-2013 - 23:00 trong Bất đẳng thức và cực trị

Bài 12: cho a,b,c,d>0 &a+b+c+d=1. CMR:
$\frac{a^{4}+b^{4}+c^{4}+d^{4}}{a^{3}+b^{3}+c^{3}+d^{3}}\geq \frac{1}{4}$

Bunyakovsky ta có: $\frac{a^4+b^4+c^4+d^4}{a^3+b^3+c^3+d^3}\geq \frac{a^3+b^3+c^3+d^3}{a^2+b^2+c^2+d^2}\geq \frac{a^2+b^2+c^2+d^2}{a+b+c+d}=a^2+b^2+c^2+d^2=\frac{1}{4}(1+1+1+1)(a^2+b^2+c^2+d^2)\geq \frac{1}{4}(a+b+c+d)^2=\frac{1}{4}$



#386559 Đề toán 9 : Câu 1 : Tìm nghiệm nguyên của phương trình $3(x^2+xy+y^2)=x+...

Đã gửi bởi triethuynhmath on 13-01-2013 - 22:50 trong Tài liệu - Đề thi

2, $x^2-3x+1+\frac{\sqrt3}{3}\sqrt{x^4+x^2+4}=0$

Ta có: $x^4+x^2+1=(x^2+x+1)(x^2-x+1)$
Đặt $a=\sqrt{x^2+x+1},b=\sqrt{x^2-x+1}\Rightarrow x^2-3x+1=2b^2-a^2$
Vậy ta được phương trình: $2b^2-a^2+\frac{\sqrt{3}}{3}ab=0$
Đây là phương trình đẳng cấp zzz...



#386557 Tìm số ngày nhiều nhất của kỳ nghỉ

Đã gửi bởi triethuynhmath on 13-01-2013 - 22:45 trong Tổ hợp và rời rạc

Đề bài: Một nhóm 10 người quyết định tham gia một kỳ nghỉ. Họ quyết định ở một khách sạn 5 sao trong suốt kì nghỉ. Do một tầng có 10 phòng nên họ đặt cả 1 lầu gồm 10 phòng được xếp theo hàng ngang. Và cứ mỗi ngày trôi qua là họ đổi phòng cho nhau theo một cách nào đó để không có 2 người nào ở cạnh nhau quá 10 ngày ( Một số người có thể ở yên một phòng trong 1 số ngày miễn là thỏa mãn yêu cầu đề bài). Tìm số ngày dài nhất của kỳ nghỉ .



#386542 Bài thực hành 1

Đã gửi bởi triethuynhmath on 13-01-2013 - 22:23 trong Nơi diễn ra Khóa học

Em xin nộp bài ạ ( Công sức 1 tiếng đồng hồ).File gửi kèm  Practice1.pdf   124K   280 Số lần tảiFile gửi kèm  Practice1.tex   5.24K   499 Số lần tải



#385959 Đề thi học sinh giỏi quận Cầu Giấy 2012 -2013

Đã gửi bởi triethuynhmath on 12-01-2013 - 17:01 trong Tài liệu - Đề thi

Bài 5 (6 điểm)
Cho đoạn thẳng $AB$ và điểm $M$ nằm giữa $A$ và B sao cho $AM <MB$ .Vẽ đường tròn $(O_1) $đường kính AM và đường trong $(O_2)$ đường kính $BM$ .$1$ tiếp tuyến chung ngoài của $2$ đường tròn tiếp xúc với $(O_1) =C$ , tiếp xúc với (O_2) =D. Gọi E là giao điểm của $AC$ và $BD$
a, c/m $EM$ cũng là tiếp tuyến chung của 2 đường tròn $(O_1)$ và $(O_2)$
b, Chứng minh $EC.EA =ED.EB$
c, Cho $AB =12cm$ và $AM =4cm$ .Vẽ $(O)$ đường kính $AB$ .Đường thẳng $CD \cap (O) =H$ và K sao cho $H \in$ cung $AK$ nhỏ .Tính đọc dài HK

Ý tưởng: Chứng minh góc $AEB=90^0$ thì lúc đó $EM$ qua trung điểm $CD$ nên là tiếp tuyến chung của 2 đường tròn.
b)Tam giác đồng dạng quá rõ, hoặc dùng hệ thức lượng trong tam giác vuông.
c) Dễ có $ OE \perp CD$. Vậy $OE$ là trung trực $HK$. Cho $OE$ cắt $HA$ tại $T$ luôn.
Có: $EA=\sqrt{AB.AM}=4\sqrt{3}$
$EM=\sqrt{EA^2-AM^2}=4\sqrt{2}$.Mà: $ET.EO=EG.EM=\frac{EM^2}{2}=16\Rightarrow ET=\frac{16}{R}=\frac{16}{6}=\frac{8}{3}\Rightarrow OT=\frac{10}{3}$
Vậy nên: $HK=2\sqrt{R^2-OT^2}=\frac{8\sqrt{14}}{3}$



#385939 Đề thi học sinh giỏi quận Cầu Giấy 2012 -2013

Đã gửi bởi triethuynhmath on 12-01-2013 - 16:29 trong Tài liệu - Đề thi

Bài 2 (4 điểm)
a, Tìm 1 số chính phương có 4 chữ , biết rằng cả 4 chữ số đều nhỏ hơn 9 và thêm vào mỗi chữ số 1 đơn vị ta được 1 số mới cũng là số chính phương .
b, Giải Phương trình $ \sqrt{3x+4} -\sqrt{11-x} +3x^2 -17x -31 =0$

a)Ý tưởng chính: Có: $\left\{\begin{matrix}
\overline{abcd}=k^2
\\ \overline{(a+1)(b+1)(c+1)(d+1)}=r^2

\end{matrix}\right.(k,r \epsilon N)\Rightarrow \left\{\begin{matrix}
\overline{abcd}=k^2
\\ 1000(a+1)+100(b+1)+10(c+1)+d+1=r^2

\end{matrix}\right.\Rightarrow \left\{\begin{matrix}
\overline{abcd}=k^2
\\ k^2+1111=r^2

\end{matrix}\right.\Rightarrow (r-k)(r+k)=1111=1111.1=11.101$
b) Điều kiện xác định: $\frac{-4}{3}\leq x \leq 11$
Ý tưởng: Nhẩm thấy nghiệm $x=7$ khiến ta nghĩ đến nhân lượng liên hợp.
Phương trình $\Leftrightarrow \sqrt{3x+4}-5-\sqrt{11-x}+2+(x-7)(3x+4)=0\Leftrightarrow \frac{3(x-7)}{\sqrt{3x+4}+5}-\frac{7-x}{\sqrt{11-x}+2}+(x-7)(3x+4)=0\Leftrightarrow \begin{bmatrix} x=7 \\\frac{3}{\sqrt{3x+4}+5}+\frac{1}{\sqrt{11-x}+2}+3x+4=0 \end{bmatrix}$
Đến đây xử lí tiếp.



#385935 Đề thi học sinh giỏi quận Cầu Giấy 2012 -2013

Đã gửi bởi triethuynhmath on 12-01-2013 - 16:18 trong Tài liệu - Đề thi

Bài 3(3 điểm)
Cho 3 số dương a,b,c Chứng minh rằng:
$\frac{a^3}{2b+3c} +\frac{b^3}{2c+3a} +\frac{c^3}{2a+3b} \geq \frac{1}{5}(a^2+b^2+c^2)$

Lớp 9 mà cho đề cũng khá được ấy chứ nhỉ, xơi câu này trước:
$Cauchy-Schwarz$ ta có: $\sum \frac{a^3}{2b+3c}=\sum \frac{a^4}{2ab+3ac}\geq \frac{(a^2+b^2+c^2)}{5(ab+bc+ca)}\geq \frac{1}{5}(a^2+b^2+c^2)$



#385932 VMO 2013 - Bài 6. Hình học

Đã gửi bởi triethuynhmath on 12-01-2013 - 16:08 trong Hình học

$ M $ di chuyển do đường thẳng qua $ H $ không phải cố định nên câu b của bạn sai rồi :(

Mình có phải là người giải đâu bạn :wacko:



#385929 VMO 2013 - Bài 6. Hình học

Đã gửi bởi triethuynhmath on 12-01-2013 - 15:36 trong Hình học

Haizzz
b) $\angle BAC=180^o-\angle BDC=\angle MPN$
$\angle MAN=\angle EHF
=\angle EAF=2\angle BAC$
Suy ra $\angle MAN=\angle MPN$ mà $\Delta MAN$ cân tại $A$ nên $P$ thuộc $(A,AM)$
Suy ra đpcm

Em giải sai rồi? $P$ đâu có nằm trên đường tròn đó?

Theo em nghĩ cách bạn này giải sai ở chỗ $M,N$ là 2 điểm di động, chỉ có $D$ là cố định nên lời giải này không chính xác vì $(A;AM)$ là đường tròn di động.



#385910 Bên lề VMO 2013

Đã gửi bởi triethuynhmath on 12-01-2013 - 13:22 trong Thi HSG Quốc gia và Quốc tế

Các anh,chị VMF thi ngày 2 thế nào rồi ạ??? Hi vọng là bội thu nhé :D :D



#385907 $\text{AD + BE + CF} \geq \frac{9\tex...

Đã gửi bởi triethuynhmath on 12-01-2013 - 13:19 trong Hình học

Cho tam giác ABC. Gọi O là giao điểm 3 đường trung trực của tam giác. AO cắt BC tại D, BO cắt AC tại E, CO cắt AB tại F. Đặt OA=OB=OC=R.
C/mR:
a) $\frac{1}{AD}+\frac{1}{BE}+\frac{1}{CF}=\frac{2}{R}$
b) $AD+BE+CF\geq \frac{9.R}{2}$

Vẽ $OM \perp BC$ tại $M$, $AH \perp BC$ tại $H$.
Ta có:
$\frac{OD}{AD}=\frac{OM}{AH}=\frac{S_{BOC}}{S_{ABC}}$
Chứng minh tương tự: $\frac{OE}{BE}=\frac{S_{OAC}}{S_{ABC}},\frac{OF}{CF}=\frac{S_{OAB}}{S_{ABC}}\Rightarrow \frac{OD}{AD}+\frac{OE}{BE}+\frac{OF}{CF}=1\Rightarrow 1-\frac{R}{AD}+1-\frac{R}{BE}+1-\frac{R}{CF}=1\Rightarrow \frac{1}{AD}+\frac{1}{BE}+\frac{1}{CF}=\frac{2}{R}$
ÁP dụng $(a+b+c)(\frac{1}{a}+\frac{1}{b}+\frac{1}{c}) \geq 9\Rightarrow đpcm$



#385474 Tìm $k$

Đã gửi bởi triethuynhmath on 10-01-2013 - 23:10 trong Số học

2.) Chứng minh rằng trong $m$ số tự nhiên bất kì bao giờ cũng có ít nhất một số chia hết cho $m$ hoặc có ít nhất một tổng chia hết cho $m$.

Xét $m$ số $a_{1},a_{2},...a_{m}$ .
Xét các tổng sau: $S_{1}=a_{1},S_{2}=a_{1}+a_{2},...S_{m}=\sum _{i=1}^{m}a_{i}$
tổng cộng có $m$ tổng.
Nếu trong $m$ tổng này có ít nhất $1$ tổng chia hết $m$ thì coi như đpcm.
Nếu như không có tổng nào chia hết $m$ thì số dư chỉ chạy từ $1->m-1$ nên theo Nguyên lí Đirichlet có ít nhất 2 tổng $S_{i},S_{j}$ có cùng số dư khi chia cho $m$.Xét hiệu của chúng chia hết cho $m$ nên có đpcm :)



#385468 $(16a+17b)(17a+16b)\vdots 11$

Đã gửi bởi triethuynhmath on 10-01-2013 - 22:54 trong Số học

1.) Cho $a,b\in \mathbb{N}$ thỏa mãn $(16a+17b)(17a+16b)\vdots 11$.
Chứng minh $(16a+17b)(17a+16b)\vdots 121$.

Do $11$ là số nguyên tố nên $\begin{bmatrix} 16a+17b \vdots 11 \\ 16b+17a \vdots 11 \end{bmatrix}$Mà: $16a+17b+17a+16b=33(a+b) \vdots 11$
Vậy cả 2 số: $16a+17b,17a+16b \vdots 11\Rightarrow đpcm$